2023 AIME II Problems/Problem 8
Revision as of 16:14, 16 February 2023 by Stevenyiweichen (talk | contribs) (Created page with "==Solution== For any <math>k\in Z</math>, we have, <cmath> \begin{align*} & \left( \omega^{3k} + \omega^k + 1 \right) \left( \omega^{3\left( 7 - k \right)} + \omega^{\left( 7...")
Solution
For any , we have, The second and the fifth equalities follow from the property that .
Therefore,
~Steven Chen (Professor Chen Education Palace, www.professorchenedu.com)